Sie sind auf Seite 1von 9

Chapter 10 Gross profit variation analysis 190

MULTIPLE CHOICE

1. Bi Corporation, which sells a single product, provided the following data from its
income statements for the calendar years, 2006 and 2005:
2006
Sales (150,000 units) P750,000
Cost of goods sold 525,000
Gross profit P225,000
2005 (Base year)
Sales (180,000 units) P720,000
Cost of goods sold 575,000
Gross profit P145,000
In an analysis of variation in gross profit between the two years, what would be the
effects of changes in sales price an sales volume?
Sales price Sales volume
A. P 150,000 favorable P 120,00 unfavorable
B. P 150,000 unfavorable P 120,000 favorable
C. P 180,000 favorable P 150,000 unfavorable
D. P 180,000 unfavorable P 150,000 favorable

1. D
? The sales price variance and the sales volume variance.
 The relevant data that could be derived from the given of the problem are:
2006 2005 Change
Quantity sold 150,000 180,000 30,000
Unit sales price P 5.00 P 4.00 P( 1.00)
Based on these data, the sales price variance and sales volume variance are
computed as follows:
Sales price variance = Change in USP x Qty. sold this year
= P(1.00) UF x 180,000 units - P(180,000) UF
Sales volume variance = Change in volume x USP last year
= 30,000 F x P5.00 = P150,000 F
A decrease in unit in sales price is unfavorable and an increase in units sold is
favorable to a firm.

2. The gross profit of MJP Company for each of the years ended December 31, 2005
and 2006, was as follows:
2005 2006
Sales P792,000 P800,000
Cost of goods sold 464,000 480,000
Gross profit P328,000 P320,000
Assuming that selling prices were 10% lower during 2005, what would be the amount
of decrease in gross profit due to the change in selling price?
A. P 8,000 C. P79,200
B. P72,000 D. P88,000 (aicpa)
Chapter 10 Gross profit variation analysis 191

2. D
? The amount of decrease in gross profit due to the change in selling price.
 Sales price variance is basically change in sales prices multiplied by the quantity sold
this year. This formula is not applicable though because the sales prices and the
quantity sold last year are not provided.
The other technique of determining the sales price variance is by getting the
difference between sales this year (STY) and sales this year at unit sales price last
year (STY@USPLY), as follows:
Sales this year P800,000
- STY @ USPLY (P800,000/90%) 888,889
Sales price variance P(88,889)
The sales this year is divided by 90% (i.e., 100% - 10%) to bring it to the level of
sales prices last year (e.g., the base, the 100%). Since prices lowered by 10%, the
sales this year is 90% that of last year’s.

3. From the records of Frank Co. the following were taken:


Quantity Sales Costs of Sales
Product Budget Actual Budget Actual Budget Actual
Green 45,000 45,800 450,000 458,000 270,000 274,800
Ann 30,000 26,700 180,000 186,900 108,000 96,120
Co 5,000 9,300 25,000 55,800 15,000 27,900
80,000 81,800 655,000 700,700 393,000 398,820
Determine the sales price (SP), sales volume (SV), cost price (CP) and cost volume
(CV) variances:
a. SP is P9,700 favorable; SV is P36,000 favorable;
CP is P5,820 favorable; and CV is P0 unfavorable
b. SP is P5,820 favorable; SV is P0 favorable;
CP is P36,700 favorable; and CV is P9,000 favorable
c. SP is P36,000 favorable; SV is P9,700 unfavorable;
CP is P0; and CV is P5,820 unfavorable
d. SP is P36,700 favorable; SV is P5,820 favorable;
CP is P0 unfavorable; and CV is P900,000 favorable (rpcpa)

3. C
? The sales price, sales volume, cost price, and cost volume variances.
 First, let us determine the units sales prices and unit cost prices.
Unit sales price Unit cost price
Actual Budget Actual Budget
Green P10.00 P10.00 P6.00 P6.00
Ann 7.00 6.00 3.60 3.60
Co 6.00 5.00 3.00 3.00

The required variances are computed as follows:


Sales price variance - ∆USP x AQ
Green = (P10 – P10) x 45,800 = P 0
Chapter 10 Gross profit variation analysis 192

Ann = (P7 – P6) x 26,700 = 26,700 UF


Co = (P6 – P5) x 9,300 = 9,300 UF P36,000 UF
Sales volume variance = ∆Q x BUSP
Green = (45,800 – 45,000) x P10 = P 8,000 UF
Ann = (26,700 – 30,000) x P6 = (19,800) F
Co = ((9,300 – 5,000) x P5 = 21,500 UF P 9,700 UF
Cost price variance = ∆UC x AQ
Green = (P6 – P6) x 45,800 = P 0
Ann = (P3.60 – P3.60) x 26,700 = 0
Co = (P3.00 – P3.00) x 9,300 = 0 P 0
Cost volume variance = ∆Q x BUC
Green = (45,800 – 45,000) x P6 = P 4,800 UF
Ann = (26,700 – 30,000) x P3.60 = (11,880) F
Co = ((9,300 – 5,000) x P3 = 12,900 UF P 5,820 UF
where: BUSP = Budgeted unit sales price.
BUC = Budgeted unit cost.
4. Mel, Inc., has a practical production capacity of two million units, the current year’s
budget was based on the production and sales of 1.4 million units during the current
year. Actual statistics came out to be: production of 1.44 million units and sales of 1.2
million. Selling price is at P20 each and the contribution margin ratio is 30%. The
peso value that best quantifies the marketing division’s failure to achieve budgeted
performance for the current year is
A. P4,800,000 unfavorable. C. P1,440,000 unfavorable.
B. P4,000,000 unfavorable. D. P1,200,000 unfavorable.

4. D
? The peso value that best quantifies the marketing division’s failure to achieve
budgeted performance for the current year.
 The actual selling price is presumed to be P20, same as the budgeted amount.
Therefore, there is no sales price variance. The required variance is the net quantity
variance.
Actual sales in units 1,200,000
- Budgeted sales in units 1,400,000
Change in quantity sold ( 200,000) UF
x Unit contribution margin (P20 x 30%) P 6
Net quantity variance P1,200,000 UF

5. In gross profit analysis, if the cost variance is zero, such variance indicates that;
A. Manufacturing management was unable to keep production costs at budgeted
costs.
B. Manufacturing management was able to control production cost below budgeted
costs.
C. Manufacturing management was able to control production cost at budgeted
costs.
D. Manufacturing management was not able to control production at budgeted costs
but purchasing was able to keep at budgeted price.

5. C
Chapter 10 Gross profit variation analysis 193

? A statement that best indicates a zero cost variance.


 A zero cost price variance indicates that there is no difference between the standard
cost prices and the actual cost prices. It also reflects the ability of the manufacturing
management to control actual unit costs in accordance with the budgeted unit costs.
Hence, choice-letter “b” is incorrect because the difference in production costs and
budgeted costs may be reflective of both cost price and cost quantity variances.
Choice-letter “d” is incorrect because production cost is affected by all inputs of
production and not by purchasing alone.

6. The differences between the master budget amounts and the amounts in the flexible
budget are due to
A. Activity level variances. C. Favorable variances
B. Gaps in affectivity. D. Unfavorable variances

6. A
? The difference between the master budget and the flexible budget.
 Master budget shows the estimated costs at a particular budgeted level of
production. Flexible budget shows the estimated costs based on a given actual level
of production. The variable costs rate and total fixed costs are the same under the
same budget levels. What differs is the estimated total variable costs. The difference
between master budget and flexible budget lies on the level of production used,
master budget uses the budgeted level while flexible budget uses the actual
production level. Choice-letter “a” is correct. The difference in the master budget
and flexible budget lies on the activity level variances.
Choice-letter “b” is incorrect because effectivity relates to the applicability or
correctness of the methods (procedures) used and therefore does not speak well of
the estimated costs of production which shall be based on the level of units
produced. Choice-letters “c” and “d” are incorrect because variances, whether
favorable or unfavorable, is the difference of actual and standards, not of two
budgeted amounts.

7. The controller of Lan Corporation found a P250,000 favorable flexible budget


revenue variance. The variance was calculated by comparing the actual results with
the flexible budget. This variance can be wholly explained by
A. The total flexible budget variance.
B. The total static budget variance.
C. Changes in unit selling prices.
D. Changes in the number of units sold.

7. A
? The one that explains the favorable flexible budget revenue variance.
 Flexible budget is estimated value based on actual capacity. Choice-letter “a” is
correct because the total flexible budget variance is explained by the net effect of the
sales price variance and the cost price variance.
Choice-letter “b” is incorrect because the total static budget variance is the net of
sales price variance, cost price variance, and quantity variance. Choice-letter “c” is
incorrect because sales price variance alone does not explain the total flexible
budget sales variance. Choice-letter “d” is incorrect because changes in the number
Chapter 10 Gross profit variation analysis 194

of units sold explains the quantity variance and is not related to the flexible budget
variance.
8. Actual and budgeted information about the sales of a product are presented below for
June:
Actual Budget
Units 8,000 10,000
Sales revenue P92,000 P105,000
The sales price variance for June was:
A. P8,000 F C. P10,000 UF
B. P10,000 F D. P10,500 UF

8. A
? The sales price variance.
 Sales price variance is the difference in unit sales price multiplied by the actual
quantity. The actual unit sales prices is P11.50 (i.e., P92,000 / 8,000) while the
budgeted unit sales price is P10.50 (i.e., P105,000 / 10,000). Therefore, the sales
price variance is P8,000 UF [i.e., (P11.50 – P10.50) x 8,000 units].

Questions 9 through 11 are based on the following information. The exhibit below
reflects a summary of performance for a single item of a retail store’s inventory for
the month ended April 30, 2006.
Flexible Static
Actual Budget Flexible (Master)
Results Variations Budget Budget
Sales (units) 11,000 - 11,000 12,000
Revenue (sales) P208,000 P12,000 U P220,000 P240,000
Variable costs 121,000 11,000 U 110,000 120,000
CM 87,000 23,000 U 110,000 120,000
Fixed costs 72,000 - 72,000 72,000
Operating income P 15,000 P23,000 U P 38,000 P 48,000

9. The sales volume variance is


A. P10,000 F C. P11,000 F
B. P10,000 U D. P12,000 U

9. B
? The sales volume variance.
 The sales volume variance refers to the contribution margin volume variance which is
the difference in actual and budgeted quantity times the budgeted unit contribution
margin of P10 (I.e., P120,000 / 12,000). Therefore, the net sales volume variance is
P(10,000) UF [i.e., (11,000 – 12,000) x P10]. The variance is unfavorable because
the actual unit sales are lower than the budgeted unit sales, a negative variance, and
is an unfavorable variance in sales variance analysis.

10. The sales price variance is:


A. P12,000 U C. P23,000 U
B. P13,000 U D. P12,000 F
Chapter 10 Gross profit variation analysis 195

10. A
? The sales price variance.
 The sales price variance is the difference between actual unit sales price and
budgeted sales price times the actual quantity sold. The actual unit sales price is
P18.91 (i.e., P208,000 / 11,000), and the budgeted unit sales price is P20 (i.e.,
P240,000 / 12,000). Therefore, the sales price variance is P12,000 U [i.e., (P18.91 –
P20.00) x 11,000 units].

11. The variable cost price variance is:


A. P11,000 U C. P23,000 U
B. P12,000 U D. P13,000 U

11. A
? The variable cost price variance.
 The variable cost price variance is the difference in actual unit variable cost and
budgeted unit variable cost times the actual quantity sold. The actual units variable
cost is P11 (i.e., P121,000 / 11,000), and the budgeted unit variable cost is P10 (i.e.,
P120,000 / 12,000). Therefore, the variable cost price variance is P11,000 U [i.e.,
{P11 – P10) x 11,000 units].

Questions 12 through 16 are based on the following information. Melanie Fashions


sells a line of women’s dresses. Melanie’s performance report for November is
shown below. The company uses a flexible budget to analyze its performance and to
measure the effect on operating income of the various factors affecting the difference
between budgeted and actual operating income.
Actual Budget
Dresses sold 5,000 6,000
Sales P 235,000 P 300,000
Variable costs (145,000) (180,000)
Contribution margin 90,000 120,000
Fixed costs ( 84,000) ( 80,000)
Operating income P 6,000 P 40,000
12. The effect of the sales quantity variance on the contribution margin for November is
A. P30,000 U C. P20,000 U
B. P18,000 U D. P15,000 U

12. C
? The effect of the sales quantity variance on the contribution margin.
 The contribution margin quantity variance is the difference in actual quantity and
budgeted quantity multiplied by the budgeted unit contribution margin of P20 (i.e.,
P120,000 / 6,000). The contribution margin quantity variance is P(20,000) U [i.e.,
{5,000 – 6,000) x P20]. The contribution margin quantity variance is the net quantity
variance of sales and variable cost.

13. The sales price variance for November is


A. P30,000 U C. P20,000 U
B. P18,000 U D. P15,000 U
Chapter 10 Gross profit variation analysis 196

13. D
? The sales price variance.
 The sales price variance is the change in actual unit sales price and budgeted unit
sales price multiplied by the actual quantity. The actual unit sales price is P47 (i.e.,
P235,000 / 5,000), and the budgeted unit sales price is P50 (i.e., P300,000 / 6,000).
As such, the sales price variance is P(15,000) U [i.e., (P47 – P50) x 5,000 units].

14. The variable cost flexible budget variance for November is


A. P5,000 F C. P4,000 F
B. P5,000 U C. P4,000 U

14. A
? The variable cost flexible budget variance.
 The variable cost flexible budget variance is P(5.000) F. It is the difference between
actual unit variable cost and budgeted unit variable cost multiplied by the actual
quantity of 5,000 units. The actual unit variable cost is P29 (i.e., P145,000 / 5,000),
and the budgeted unit variable cost is P30 (i.e., P180,000 / 6,000).

15. The fixed cost variance for November is


A. P5,000 F C. P4,000 F
B. P5,000 U D. P4,000 U

15. D
? The fixed cost variance.
 The fixed cost variance is the difference in actual fixed costs and budgeted fixed cost.
Therefore, the fixed cost variance is P4,000 U (i.e., P84,000 – P80,000).

16. What additional information is needed for Melanie to calculate the peso impact of a
change in market share on operating income for November?
A. Melanie’s budgeted market share and the budgeted total market size.
B. Melanie’s budgeted market share, the budgeted total market size, and average
market selling price.
C. Melanie’s budgeted market share and the actual total market size.
D. Melanie’s actual market share and the actual total market size.

16. C
? The additional informational needed to calculate the peso impact of a change in
market share on operating income.
 The market share variance is determined by getting the difference in actual market
share and the budgeted market share multiplied by the budgeted unit sales price.
Hence, choice-letter “c” is correct.
Choice-letters “a” and “b” are incorrect because they do not provide the actual
market size or actual market share. Choice-letter “d” is incorrect because it does not
provide the budgeted market share.

Questions 17 and 18 are based on the following information. Franklin, Inc.,


manufactures and sells boxes of pocket protectors. The static master budget and the
actual results for May 2006 appear below:
Chapter 10 Gross profit variation analysis 197

Actual Static Budget


Unit sales 12,000 10,000
Sales P132,000 P100,000
Variable costs of sales 70,800 60,000
CM 61,200 40,000
Fixed costs 32,000 30,000
Operating income P 29,200 P 10,000
17. The operating income for Franklin using a flexible budget for May 2006 is
A. P12,000 C. P30,000
B. P19,200 D. P18,000

17. D
? The flexible budget operating income.
 Flexible budget operating income is estimated contribution margin based on actual
units sold less budgeted fixed costs. The budgeted unit contribution margin is P4
(i.e., P40,000 / 10,000). Therefore, the flexible budget operating income is P18,000.
computed as follows:
Estimated contribution margin (P4 x 12,000) P48,000
- Budgeted fixed costs 30,000
Flexible budget operating income P18,000

18. Which one of the following statements concerning Franklin’s actual results for May
2006 is correct?
a. The flexible budget variance is P8,000 favorable.
b. The sales price variance is P32,000 favorable.
c. The sales volume variance is P8,000 favorable.
d. The fixed costs flexible budget variance is P4,000 favorable.

18. C
? The correct variance with respect to May 2005 operations.
 Choice-letter “c” is the correct answer. The sales volume variance of P8,000 F is
determined by multiplying the change in unit sold (e.g., 2,000 units) by the budgeted
unit contribution margin of P4.
Choice-letter “a” is incorrect because the flexible budget variance is P11,200 F
(i.e., P29,200 – P18,000). Choice-letter “b” is incorrect because the sales price
variance is P12,000 F [(P11 – P10) x 12,000 units]. Choice-letter “d” is also incorrect
because the fixed costs flexible budget variance is P2,000 (i.e., P32,000 – P30,000)

Sales variances
19. The sales volume variance equals
A. A flexible budget amount minus a static budget amount.
B. Actual operating income minus flexible budget operating income.
C. Actual unit price minus budgeted unit price, times the actual units produced.
D. Budgeted unit price times the difference between actual inputs and budgeted
inputs for the actual activity level achieved. (ing)
Chapter 10 Gross profit variation analysis 198

19. A
? The sales volume variance.
 Sales volume variance refers to the change between budgeted sales and actual
sales activity levels. It further assumes that unit sales price, unit variable costs, and
fixed costs are constant. Since flexible budget is based on actual capacity and static
budget is based on budgeted capacity, their difference is therefore the sales volume
variance.
Choice-letter “b” is incorrect because sales volume variance does not refer to
changes in operating income. Choice-letter “c’ is incorrect because it refers to
production price variance. Choice-letter “d” is incorrect because it refers to efficiency
variance (e.g., budgeted inputs for the actual capacity level is the standard capacity),
and the term “inputs” here refers to hours.

20. In analyzing company operations, the controller of the FM Corporation found a


P250,000 favorable flexible budget revenue variance. The variance was calculated
by comparing the actual results with the flexible budget. This variance can be wholly
explained by
A. The total flexible budget variance.
B. The total static budget variance.
C. Changes in unit selling prices.
D. Changes in the number of units sold. (cma)

20. C
? An explanation for a favorable flexible budget revenue variance.
 Actual cost is actual hours at actual price per hour. Flexible budget is based on
standard hours which is actual production multiplied by the standard hours per unit.
Therefore, the difference between actual cost and flexible budget is price variance.
Translating this variance to revenue means it is a sales price variance, or change in
unit sales prices. Choice-letter “c” is correct.
Choice-letter “a” is incorrect because flexible budget variance covers items other
than revenue. Choice-letter “b” is incorrect because total static budget variance
includes price and volume variance and, also, it does not only refer to revenue.
Choice-letter “d” is incorrect because changes in the number of units sold is volume
variance which is the difference between flexible budget and static budget.

 end 

Das könnte Ihnen auch gefallen